Đến nội dung

xuanhoan23112002 nội dung

Có 95 mục bởi xuanhoan23112002 (Tìm giới hạn từ 11-05-2020)



Sắp theo                Sắp xếp  

#642083 $\boxed{\text{Chuyên Đề}}$ Phương trình vô tỉ - Hệ phương...

Đã gửi bởi xuanhoan23112002 on 25-06-2016 - 09:09 trong Phương trình, hệ phương trình và bất phương trình

giúp mình bài này với: $x^4+1=5x(x^2-2)$




#662648 $\boxed{\text{Chuyên Đề}}$ Phương trình vô tỉ - Hệ phương...

Đã gửi bởi xuanhoan23112002 on 21-11-2016 - 20:49 trong Phương trình, hệ phương trình và bất phương trình

Các bài làm hầu như không ghi ĐKXĐ vào, các bạn chú ý nhé. (Nhớ trích dẫn đề nữa)

 

Giải pt:

19) $\sqrt{2x^2+8x+6}+\sqrt{x^2-1}=2x+2$

 

20) $\sqrt[3]{24+x}+\sqrt{12-x}=6$

 

21) $(x+1)(x+4)=5\sqrt{x^2+5x+28}$

 

22) $4\sqrt{(4-x)(2+x)}=x^2-2x-12$

 

23) $\sqrt{3x-2}+\sqrt{x-1}=4x-9+2\sqrt{3x^2-5x+2}$

 

24) $5\sqrt{x}+\frac{5}{2\sqrt{x}}=2x+\frac{1}{2x}+4$

 

25) $x^2-2x=2\sqrt{2x-1}$

 

26) $x^3+1=2\sqrt[3]{2x-1}$

câu 21:DKXD: với mọi x

sau đó đặt cận vệ phải là a(a>=0) tôi biểu diễn về trại theo a và phân tích thành tích là xong




#668372 $\boxed{\text{Chuyên Đề}}$ Phương trình vô tỉ - Hệ phương...

Đã gửi bởi xuanhoan23112002 on 15-01-2017 - 10:34 trong Phương trình, hệ phương trình và bất phương trình

Giải phương trình này: $\sqrt{1-\frac{1}{x}}+\sqrt{x^2-x}=2$




#662621 $\boxed{\text{Chuyên Đề}}$ Phương trình vô tỉ - Hệ phương...

Đã gửi bởi xuanhoan23112002 on 21-11-2016 - 17:24 trong Phương trình, hệ phương trình và bất phương trình

Các bạn suy nghĩ bài này xem:

giải hệ phương trình:

     5x2+ 2y2+ z=2

     xy+ yz+ xz =1




#662624 $\boxed{\text{Chuyên Đề}}$ Phương trình vô tỉ - Hệ phương...

Đã gửi bởi xuanhoan23112002 on 21-11-2016 - 17:36 trong Phương trình, hệ phương trình và bất phương trình

Giải phương trình  x2 - 5x + 36 = 8 $\sqrt{3x+4}$

DKXD:x>=-4/3

Cậu bình phương cả 2 vế lên:

(x2-5x+36)2=64(3x+4)

x4-10x3+ 97x- 552x+1040=0

(x-4)2(x- 2x + 65)=0

x=4 (thỏa man DKXD)




#705432 $\frac{1}{a^2+2b^2+3}+\frac{1}...

Đã gửi bởi xuanhoan23112002 on 10-04-2018 - 21:59 trong Bất đẳng thức và cực trị

Từ giả thiết suy ra abc<=1

a2+2b2+3>=2ab+2b+2>0

$\frac{1}{a2+2b2+3}$<=$\frac{1}{2ab+2b+2}$

Làm tương tự như trên với các phân thức còn lại cùng với abc<=1 ta có điều phải chứng minh




#712421 $2^n-1$ là số nguyên tố

Đã gửi bởi xuanhoan23112002 on 12-07-2018 - 22:34 trong Số học

Bài này dùng phản chứng thôi.




#709874 $a^2 + b^2 + c^2 + abc = 4$

Đã gửi bởi xuanhoan23112002 on 04-06-2018 - 07:45 trong Bất đẳng thức và cực trị

Bạn có thể tham khảo tại đây: http://diendantoanho...2b2c2abc-geq-4/

Đáp án: $Min P=2\Leftrightarrow (a,b,c)=(2,0,0)$ và các hoán vị của nó

             $Max P=3\Leftrightarrow (a,b,c)=(1,1,1)$




#709905 $a^2 + b^2 + c^2 + abc = 4$

Đã gửi bởi xuanhoan23112002 on 04-06-2018 - 12:38 trong Bất đẳng thức và cực trị

Tìm max: http://diendantoanho...-định-năm-2018/ (chỉ việc thay mỗi số 2 thành số 1 thôi a trình bày đầy đủ rồi)

Tìm min:

Nếu cả 3 số a, b, c đều > 2 hiển nhiên suy ra điều vô lí

Do đó ta giả sử: $c\leq 2$ nên $abc\leq 2ab$

$\Rightarrow 4=a^2+b^2+c^2+2abc\leq a^2+b^2+c^2+2ab=(a+b)^2+c^2\leq (a+b+c)^2$

$\Rightarrow a+b+c\geq 2$




#711677 $f(x)=ax^4+bx+c> 0 \forall x> 0$

Đã gửi bởi xuanhoan23112002 on 27-06-2018 - 16:55 trong Đa thức

Cho $a\neq 0$ và $f(x)=ax^4+bx+c> 0 \forall x> 0$

CMR: $f(x)$ được biểu diễn ở dạng tổng bình phương của 2 tam thức bậc hai.




#691945 $P=\frac{(x+1)^{2}(y+1)^{2}}{(x-...

Đã gửi bởi xuanhoan23112002 on 31-08-2017 - 13:28 trong Bất đẳng thức và cực trị

Bài này khá dễ giống đề thi lớp 10 ở VĨnh Phúc,. Bạn có thể tham khảo, hãy like cho mình nhé




#704021 2(ab+bc+ca)+$\frac{1}{ab}+\frac{1...

Đã gửi bởi xuanhoan23112002 on 20-03-2018 - 22:08 trong Bất đẳng thức và cực trị

bạn bị ngược dấu hay sao ấy

nguoc dau cho nao




#703960 2(ab+bc+ca)+$\frac{1}{ab}+\frac{1...

Đã gửi bởi xuanhoan23112002 on 19-03-2018 - 23:21 trong Bất đẳng thức và cực trị

Su dung pp pqr
Dat p=a+b+c=3

q=ab+bc+ca

r=abc,r<=1

BDT tuong duong 2q+3/r>=9

Hay 2qr+3>=9r

Ma q>=3*can(r)( do q^2>=3pr)

Dua ve bpt an r giai voi chu y r<=1




#688311 58th IMO 2017

Đã gửi bởi xuanhoan23112002 on 22-07-2017 - 08:38 trong Thi HSG Quốc gia và Quốc tế

moi nguoi nghi ra huong giai quyet bai 3 chua




#706779 [TOPIC] SỐ HỌC ÔN TẬP THPT CHUYÊN TOÁN 10 NĂM HỌC 2018-2019

Đã gửi bởi xuanhoan23112002 on 23-04-2018 - 21:32 trong Tài liệu - Đề thi

Bài 94: Giải phương trình nghiệm nguyên: $x^3-x^2+8=y^2$




#706771 [TOPIC] SỐ HỌC ÔN TẬP THPT CHUYÊN TOÁN 10 NĂM HỌC 2018-2019

Đã gửi bởi xuanhoan23112002 on 23-04-2018 - 20:09 trong Tài liệu - Đề thi

Bài 91: Tìm tất cả các cặp số (p, n) với p là số nguyên tố và n là số nguyên dương thỏa mãn:

$p^3-2p^2+p+1=3^n$




#706634 [TOPIC] SỐ HỌC ÔN TẬP THPT CHUYÊN TOÁN 10 NĂM HỌC 2018-2019

Đã gửi bởi xuanhoan23112002 on 22-04-2018 - 09:04 trong Tài liệu - Đề thi

Bài 73: Ta chứng minh bài toán bằng phương pháp phản chứng

Thật vậy ta có thể giả sử a+b là số nguyên tố 

Theo giả thiết ta có: $(a+b)(b+c)(c+a)-8abc \vdots a+b$

Hay $8abc \vdots a+b$. Lại có a+b là số lẻ nên gcd(a+b,8)=1

Do đó $abc \vdots a+b$ 

Mà a+b là số nguyên tố nên xảy ra 1 trong 3 trường hợp sau: $a \vdots a+b$ hoặc $b \vdots a+b$ hoặc $c \vdots a+b$ (điều này là vô lí do a, b, c là 3 cạnh của tam giác nên max{a, b, c}< a+b)

Nên ta có điều giả sử là sai.

Vậy a+b phải là số nguyên tố




#706679 [TOPIC] SỐ HỌC ÔN TẬP THPT CHUYÊN TOÁN 10 NĂM HỌC 2018-2019

Đã gửi bởi xuanhoan23112002 on 22-04-2018 - 20:43 trong Tài liệu - Đề thi

Bài 76: Xét 2 trường hợp

Nếu $p\geq q$ từ giả thiết suy ra $q\leq 3$. Mà q là số nguyên tố nên q thuộc{2; 3}. Thử trực tiếp ta thu được (p, q)=(3, 3)

Nếu $p\leq q$ từ giả thiết suy ra $p\leq 5$. Mà p là số nguyên tố nên p thuộc{2, 3, 5}. Thử trực tiếp ta thu được (p, q)=(3, 3)

Vậy cặp số (p, q) thỏa mãn bài là (3, 3)

Bài 79: Gợi ý sử dụng nguyên lí cực hạn. ĐS: p=2 hoặc p=3




#706251 [TOPIC] SỐ HỌC ÔN TẬP THPT CHUYÊN TOÁN 10 NĂM HỌC 2018-2019

Đã gửi bởi xuanhoan23112002 on 17-04-2018 - 23:15 trong Tài liệu - Đề thi

Lời giải của mình cho bài 35 như sau:

PT đã cho $\Leftrightarrow y^3=(x^3+1)(x^2+1)$

Do x là số lẻ ta dễ dàng chứng minh được gcd(x3+1,x2+1)=1

$\Rightarrow$ x3+1 là lập phương của 1 số nguyên.

Như vậy, x3 và x3+1 là 2 số nguyên liên tiếp và đều là lập phương của các số nguyên, và theo giả thiết x là số lẻ nên suy ra x= -1

Từ đó thay vào giả thiết tìm được y= 0

Vậy cặp số (x, y) thỏa mãn bài là (0, -1)




#706727 [TOPIC] SỐ HỌC ÔN TẬP THPT CHUYÊN TOÁN 10 NĂM HỌC 2018-2019

Đã gửi bởi xuanhoan23112002 on 23-04-2018 - 13:14 trong Tài liệu - Đề thi

Bài 85(VMO 2007): Cho x, y là các số nguyên, $x\neq -1, y\neq -1$ thoả mãn: $\frac{x^4-1}{y+1}+\frac{y^4-1}{x+1}$ là số nguyên. CMR

$x^{4}y^{44}-1$ chia hết cho x+1




#706687 [TOPIC] SỐ HỌC ÔN TẬP THPT CHUYÊN TOÁN 10 NĂM HỌC 2018-2019

Đã gửi bởi xuanhoan23112002 on 22-04-2018 - 21:15 trong Tài liệu - Đề thi

Bài 82: Tìm tất cả các cặp số nguyên tố (p, q) thỏa mãn $p> q$ và $p^3-q^7=p-q$




#706708 [TOPIC] SỐ HỌC ÔN TẬP THPT CHUYÊN TOÁN 10 NĂM HỌC 2018-2019

Đã gửi bởi xuanhoan23112002 on 22-04-2018 - 23:08 trong Tài liệu - Đề thi

Bài 83: Giải phương trình nghiệm nguyên dương:

$x^3-(x+y+z)^2=(y+z)^3+34$




#712416 Đề thi IMO 2018

Đã gửi bởi xuanhoan23112002 on 12-07-2018 - 21:52 trong Thi HSG Quốc gia và Quốc tế

Kết quả chính thức IMO 2018



#709817 Đề thi lớp 10 môn Toán vào Trường THPT Chuyên Lam Sơn

Đã gửi bởi xuanhoan23112002 on 03-06-2018 - 10:57 trong Tài liệu - Đề thi

Câu 5: Bất đẳng thức đã cho tương đương với:

$a^2b+b^2c+c^2a+ab^2+bc^2+ca^2> a^3+b^3+c^3+2abc$

$\Leftrightarrow a(b-c)^2+b^2(c+a-b)+c^2(a+b-c)> 0$ (luôn đúng đó a, b, c là độ dài 3 cạnh của 1 tam giác)




#685108 Đề thi toán chuyên - chuyên KHTN ĐHQG HÀ Nội vòng 2 2017

Đã gửi bởi xuanhoan23112002 on 20-06-2017 - 09:12 trong Tài liệu - Đề thi

Câu II 2.

Từ giả thiết ta có được $\frac{1}{(a+1)(b+1)}+\frac{1}{(c+1)(b+1)}+\frac{1}{(a+1)(c+1)}=1$

 Đặt $a+1=\frac{\sqrt{3}}{x}, b+1=\frac{\sqrt{3}}{y},c+1=\frac{\sqrt{3}}{z}$

Giả thiết trở thành $xy+yz+zx=3$ và

$P= \sqrt{3} ( \frac{1}{\frac{3}{x}+x} +\frac{1}{\frac{3}{y}+y} +\frac{1}{\frac{3}{z}+z})$

   $= \sqrt{3} (\frac{x}{x^{2}+3}+\frac{y}{y^{2}+3}+\frac{z}{z^{2}+3})$

Sử dụng giả thiết ta có

  $P=\sqrt{3}( \frac{x}{(x+y)(x+z)}+ \frac{y}{(x+y)(y+z)}+ \frac{z}{(z+y)(x+z)})$

    $=\sqrt{3}( \frac{2(xy+yz+zx)}{(x+y)(y+z)(z+x)})$

Mặt khác $(x+y)(y+z)(z+x) \geq \frac{8}{9}(x+y+z)(xy+yz+zx) \geq \frac{8}{3}(xy+yz+zx)$

Suy ra $P \leq \sqrt{3}\frac{3}{4}= \frac{3\sqrt{3}}{4}$

Đẳng thức xảy ra khi $a=b=c = \sqrt{3}-1$                                                      

mình có cách khác nhanh hơn đặt a+1=x;b+1=y;c+1=z thì bài toán sẽ tự nhiên hơn và ứng dụng được cả giả thiết